Proof $y=x^3/(x^2+1)$ is injective












0












$begingroup$


i need to proof that $$f(x)=frac{x^3}{x^2+1}$$ is injective



I tried $f(x_1)=f(x_2)$ and I also tried with Rolle's theorem.










share|cite|improve this question











$endgroup$

















    0












    $begingroup$


    i need to proof that $$f(x)=frac{x^3}{x^2+1}$$ is injective



    I tried $f(x_1)=f(x_2)$ and I also tried with Rolle's theorem.










    share|cite|improve this question











    $endgroup$















      0












      0








      0





      $begingroup$


      i need to proof that $$f(x)=frac{x^3}{x^2+1}$$ is injective



      I tried $f(x_1)=f(x_2)$ and I also tried with Rolle's theorem.










      share|cite|improve this question











      $endgroup$




      i need to proof that $$f(x)=frac{x^3}{x^2+1}$$ is injective



      I tried $f(x_1)=f(x_2)$ and I also tried with Rolle's theorem.







      functions






      share|cite|improve this question















      share|cite|improve this question













      share|cite|improve this question




      share|cite|improve this question








      edited Dec 16 '18 at 14:05









      egreg

      184k1486205




      184k1486205










      asked Dec 16 '18 at 14:00









      shayshay

      103




      103






















          4 Answers
          4






          active

          oldest

          votes


















          1












          $begingroup$

          Assume $exists a,b: f(a)=f(b)$, then begin{align*}frac{a^3}{a^2+1}=frac{b^3}{b^2+1}&iff a^3b^2+a^3=b^3a^2+b^3 \ &iff a^3b^2-a^2b^3+a^3-b^3=0 \&iff a^2b^2(a-b)+(a-b)(a^2-ab+b^2)=0 \ &iff (a-b)left[a^2b^2+a^2+ab+b^2right]=0\&iff a=bvee a^2b^2+a^2+ab+b^2=0end{align*}



          Edit: based on @egreg's comment: $$a^2b^2+a^2+ab+b^2=(ab)^2+left(a+frac{b}{2}right)^2+frac{3b^2}{4}ge 0$$ and equality holds if and only if $a=b=0$.



          Either way, we found that $a=b$, hence the function is injective.






          share|cite|improve this answer











          $endgroup$









          • 2




            $begingroup$
            $a^2b^2+a^2+ab+b^2=(ab)^2+(a+b/2)^2+3b^2/4$. A sum of squares is nonnegative and is zero if and only if each term is zero.
            $endgroup$
            – egreg
            Dec 16 '18 at 14:46










          • $begingroup$
            @egreg, thanks, I have looked after the factorization, but didn't manage to compute it by myself.
            $endgroup$
            – Galc127
            Dec 16 '18 at 14:48



















          3












          $begingroup$

          Hint: Prove that, for each real $x$, $f'(x)geqslant0$.






          share|cite|improve this answer









          $endgroup$













          • $begingroup$
            I did it, but still dont know how it's can help me
            $endgroup$
            – shay
            Dec 16 '18 at 14:08






          • 1




            $begingroup$
            SInce $f'(x)$ is always non-negative, $f$ is invreasing. Now, suppose that $f(x)=f(y)$ for some $x$ and some $y$ such that $x<y$. Then $f$ is constant in $[x,y]$ and therefore $f'$ is $0$ there. But $f'$ only has one single zero.
            $endgroup$
            – José Carlos Santos
            Dec 16 '18 at 14:10



















          2












          $begingroup$

          Here is an approach if you want to avoid derivatives. For $x neq 0$, we have
          $$frac{x^3}{x^2 + 1} = frac{1}{frac{1}{x} + frac{1}{x^3}}.$$



          The function $1/x + 1/x^3$ for $x > 0$ is the sum of two strictly decreasing functions, hence itself strictly decreasing and therefore one to one with range $(0, infty)$. Hence $1/(1/x + 1/x^3)$ is strictly increasing and one to one with range $(0, infty)$. For $x < 0$, notice $1/x + 1/x^3$ is an odd function, and therefore a similar argument shows $1/(1/x + 1/x^3)$ is one to one with range $(-infty, 0)$.



          In fact, this shows that if you add back the point $x = 0$, the function is a bijection since the function is zero here.






          share|cite|improve this answer









          $endgroup$





















            1












            $begingroup$

            Suppose $f(a)=f(b)$: then
            $$
            a^3(b^2+1)=b^3(a^2+1)
            $$

            This implies
            $$
            a^3b^2-a^2b^3+a^3-b^3=0
            $$

            and so
            $$
            a^2b^2(a-b)+(a-b)(a^2+ab+b^2)=0
            $$

            Conclude.




            If you know that $a^2+ab+b^2le0$ if and only if $a=b=0$, you're done.




            With calculus,
            $$
            f'(x)=frac{3x^2(x^2+1)-2x^4}{(x^2+1)^2}
            $$

            The derivative only vanishes at $0$, so Rolle's theorem would only apply to an interval of the form $[a,b]$, with $a<0$ and $b>0$. Can $f(a)=f(b)$ in this case?






            share|cite|improve this answer









            $endgroup$













              Your Answer





              StackExchange.ifUsing("editor", function () {
              return StackExchange.using("mathjaxEditing", function () {
              StackExchange.MarkdownEditor.creationCallbacks.add(function (editor, postfix) {
              StackExchange.mathjaxEditing.prepareWmdForMathJax(editor, postfix, [["$", "$"], ["\\(","\\)"]]);
              });
              });
              }, "mathjax-editing");

              StackExchange.ready(function() {
              var channelOptions = {
              tags: "".split(" "),
              id: "69"
              };
              initTagRenderer("".split(" "), "".split(" "), channelOptions);

              StackExchange.using("externalEditor", function() {
              // Have to fire editor after snippets, if snippets enabled
              if (StackExchange.settings.snippets.snippetsEnabled) {
              StackExchange.using("snippets", function() {
              createEditor();
              });
              }
              else {
              createEditor();
              }
              });

              function createEditor() {
              StackExchange.prepareEditor({
              heartbeatType: 'answer',
              autoActivateHeartbeat: false,
              convertImagesToLinks: true,
              noModals: true,
              showLowRepImageUploadWarning: true,
              reputationToPostImages: 10,
              bindNavPrevention: true,
              postfix: "",
              imageUploader: {
              brandingHtml: "Powered by u003ca class="icon-imgur-white" href="https://imgur.com/"u003eu003c/au003e",
              contentPolicyHtml: "User contributions licensed under u003ca href="https://creativecommons.org/licenses/by-sa/3.0/"u003ecc by-sa 3.0 with attribution requiredu003c/au003e u003ca href="https://stackoverflow.com/legal/content-policy"u003e(content policy)u003c/au003e",
              allowUrls: true
              },
              noCode: true, onDemand: true,
              discardSelector: ".discard-answer"
              ,immediatelyShowMarkdownHelp:true
              });


              }
              });














              draft saved

              draft discarded


















              StackExchange.ready(
              function () {
              StackExchange.openid.initPostLogin('.new-post-login', 'https%3a%2f%2fmath.stackexchange.com%2fquestions%2f3042639%2fproof-y-x3-x21-is-injective%23new-answer', 'question_page');
              }
              );

              Post as a guest















              Required, but never shown

























              4 Answers
              4






              active

              oldest

              votes








              4 Answers
              4






              active

              oldest

              votes









              active

              oldest

              votes






              active

              oldest

              votes









              1












              $begingroup$

              Assume $exists a,b: f(a)=f(b)$, then begin{align*}frac{a^3}{a^2+1}=frac{b^3}{b^2+1}&iff a^3b^2+a^3=b^3a^2+b^3 \ &iff a^3b^2-a^2b^3+a^3-b^3=0 \&iff a^2b^2(a-b)+(a-b)(a^2-ab+b^2)=0 \ &iff (a-b)left[a^2b^2+a^2+ab+b^2right]=0\&iff a=bvee a^2b^2+a^2+ab+b^2=0end{align*}



              Edit: based on @egreg's comment: $$a^2b^2+a^2+ab+b^2=(ab)^2+left(a+frac{b}{2}right)^2+frac{3b^2}{4}ge 0$$ and equality holds if and only if $a=b=0$.



              Either way, we found that $a=b$, hence the function is injective.






              share|cite|improve this answer











              $endgroup$









              • 2




                $begingroup$
                $a^2b^2+a^2+ab+b^2=(ab)^2+(a+b/2)^2+3b^2/4$. A sum of squares is nonnegative and is zero if and only if each term is zero.
                $endgroup$
                – egreg
                Dec 16 '18 at 14:46










              • $begingroup$
                @egreg, thanks, I have looked after the factorization, but didn't manage to compute it by myself.
                $endgroup$
                – Galc127
                Dec 16 '18 at 14:48
















              1












              $begingroup$

              Assume $exists a,b: f(a)=f(b)$, then begin{align*}frac{a^3}{a^2+1}=frac{b^3}{b^2+1}&iff a^3b^2+a^3=b^3a^2+b^3 \ &iff a^3b^2-a^2b^3+a^3-b^3=0 \&iff a^2b^2(a-b)+(a-b)(a^2-ab+b^2)=0 \ &iff (a-b)left[a^2b^2+a^2+ab+b^2right]=0\&iff a=bvee a^2b^2+a^2+ab+b^2=0end{align*}



              Edit: based on @egreg's comment: $$a^2b^2+a^2+ab+b^2=(ab)^2+left(a+frac{b}{2}right)^2+frac{3b^2}{4}ge 0$$ and equality holds if and only if $a=b=0$.



              Either way, we found that $a=b$, hence the function is injective.






              share|cite|improve this answer











              $endgroup$









              • 2




                $begingroup$
                $a^2b^2+a^2+ab+b^2=(ab)^2+(a+b/2)^2+3b^2/4$. A sum of squares is nonnegative and is zero if and only if each term is zero.
                $endgroup$
                – egreg
                Dec 16 '18 at 14:46










              • $begingroup$
                @egreg, thanks, I have looked after the factorization, but didn't manage to compute it by myself.
                $endgroup$
                – Galc127
                Dec 16 '18 at 14:48














              1












              1








              1





              $begingroup$

              Assume $exists a,b: f(a)=f(b)$, then begin{align*}frac{a^3}{a^2+1}=frac{b^3}{b^2+1}&iff a^3b^2+a^3=b^3a^2+b^3 \ &iff a^3b^2-a^2b^3+a^3-b^3=0 \&iff a^2b^2(a-b)+(a-b)(a^2-ab+b^2)=0 \ &iff (a-b)left[a^2b^2+a^2+ab+b^2right]=0\&iff a=bvee a^2b^2+a^2+ab+b^2=0end{align*}



              Edit: based on @egreg's comment: $$a^2b^2+a^2+ab+b^2=(ab)^2+left(a+frac{b}{2}right)^2+frac{3b^2}{4}ge 0$$ and equality holds if and only if $a=b=0$.



              Either way, we found that $a=b$, hence the function is injective.






              share|cite|improve this answer











              $endgroup$



              Assume $exists a,b: f(a)=f(b)$, then begin{align*}frac{a^3}{a^2+1}=frac{b^3}{b^2+1}&iff a^3b^2+a^3=b^3a^2+b^3 \ &iff a^3b^2-a^2b^3+a^3-b^3=0 \&iff a^2b^2(a-b)+(a-b)(a^2-ab+b^2)=0 \ &iff (a-b)left[a^2b^2+a^2+ab+b^2right]=0\&iff a=bvee a^2b^2+a^2+ab+b^2=0end{align*}



              Edit: based on @egreg's comment: $$a^2b^2+a^2+ab+b^2=(ab)^2+left(a+frac{b}{2}right)^2+frac{3b^2}{4}ge 0$$ and equality holds if and only if $a=b=0$.



              Either way, we found that $a=b$, hence the function is injective.







              share|cite|improve this answer














              share|cite|improve this answer



              share|cite|improve this answer








              edited Dec 16 '18 at 14:50

























              answered Dec 16 '18 at 14:16









              Galc127Galc127

              3,3121334




              3,3121334








              • 2




                $begingroup$
                $a^2b^2+a^2+ab+b^2=(ab)^2+(a+b/2)^2+3b^2/4$. A sum of squares is nonnegative and is zero if and only if each term is zero.
                $endgroup$
                – egreg
                Dec 16 '18 at 14:46










              • $begingroup$
                @egreg, thanks, I have looked after the factorization, but didn't manage to compute it by myself.
                $endgroup$
                – Galc127
                Dec 16 '18 at 14:48














              • 2




                $begingroup$
                $a^2b^2+a^2+ab+b^2=(ab)^2+(a+b/2)^2+3b^2/4$. A sum of squares is nonnegative and is zero if and only if each term is zero.
                $endgroup$
                – egreg
                Dec 16 '18 at 14:46










              • $begingroup$
                @egreg, thanks, I have looked after the factorization, but didn't manage to compute it by myself.
                $endgroup$
                – Galc127
                Dec 16 '18 at 14:48








              2




              2




              $begingroup$
              $a^2b^2+a^2+ab+b^2=(ab)^2+(a+b/2)^2+3b^2/4$. A sum of squares is nonnegative and is zero if and only if each term is zero.
              $endgroup$
              – egreg
              Dec 16 '18 at 14:46




              $begingroup$
              $a^2b^2+a^2+ab+b^2=(ab)^2+(a+b/2)^2+3b^2/4$. A sum of squares is nonnegative and is zero if and only if each term is zero.
              $endgroup$
              – egreg
              Dec 16 '18 at 14:46












              $begingroup$
              @egreg, thanks, I have looked after the factorization, but didn't manage to compute it by myself.
              $endgroup$
              – Galc127
              Dec 16 '18 at 14:48




              $begingroup$
              @egreg, thanks, I have looked after the factorization, but didn't manage to compute it by myself.
              $endgroup$
              – Galc127
              Dec 16 '18 at 14:48











              3












              $begingroup$

              Hint: Prove that, for each real $x$, $f'(x)geqslant0$.






              share|cite|improve this answer









              $endgroup$













              • $begingroup$
                I did it, but still dont know how it's can help me
                $endgroup$
                – shay
                Dec 16 '18 at 14:08






              • 1




                $begingroup$
                SInce $f'(x)$ is always non-negative, $f$ is invreasing. Now, suppose that $f(x)=f(y)$ for some $x$ and some $y$ such that $x<y$. Then $f$ is constant in $[x,y]$ and therefore $f'$ is $0$ there. But $f'$ only has one single zero.
                $endgroup$
                – José Carlos Santos
                Dec 16 '18 at 14:10
















              3












              $begingroup$

              Hint: Prove that, for each real $x$, $f'(x)geqslant0$.






              share|cite|improve this answer









              $endgroup$













              • $begingroup$
                I did it, but still dont know how it's can help me
                $endgroup$
                – shay
                Dec 16 '18 at 14:08






              • 1




                $begingroup$
                SInce $f'(x)$ is always non-negative, $f$ is invreasing. Now, suppose that $f(x)=f(y)$ for some $x$ and some $y$ such that $x<y$. Then $f$ is constant in $[x,y]$ and therefore $f'$ is $0$ there. But $f'$ only has one single zero.
                $endgroup$
                – José Carlos Santos
                Dec 16 '18 at 14:10














              3












              3








              3





              $begingroup$

              Hint: Prove that, for each real $x$, $f'(x)geqslant0$.






              share|cite|improve this answer









              $endgroup$



              Hint: Prove that, for each real $x$, $f'(x)geqslant0$.







              share|cite|improve this answer












              share|cite|improve this answer



              share|cite|improve this answer










              answered Dec 16 '18 at 14:05









              José Carlos SantosJosé Carlos Santos

              167k22132235




              167k22132235












              • $begingroup$
                I did it, but still dont know how it's can help me
                $endgroup$
                – shay
                Dec 16 '18 at 14:08






              • 1




                $begingroup$
                SInce $f'(x)$ is always non-negative, $f$ is invreasing. Now, suppose that $f(x)=f(y)$ for some $x$ and some $y$ such that $x<y$. Then $f$ is constant in $[x,y]$ and therefore $f'$ is $0$ there. But $f'$ only has one single zero.
                $endgroup$
                – José Carlos Santos
                Dec 16 '18 at 14:10


















              • $begingroup$
                I did it, but still dont know how it's can help me
                $endgroup$
                – shay
                Dec 16 '18 at 14:08






              • 1




                $begingroup$
                SInce $f'(x)$ is always non-negative, $f$ is invreasing. Now, suppose that $f(x)=f(y)$ for some $x$ and some $y$ such that $x<y$. Then $f$ is constant in $[x,y]$ and therefore $f'$ is $0$ there. But $f'$ only has one single zero.
                $endgroup$
                – José Carlos Santos
                Dec 16 '18 at 14:10
















              $begingroup$
              I did it, but still dont know how it's can help me
              $endgroup$
              – shay
              Dec 16 '18 at 14:08




              $begingroup$
              I did it, but still dont know how it's can help me
              $endgroup$
              – shay
              Dec 16 '18 at 14:08




              1




              1




              $begingroup$
              SInce $f'(x)$ is always non-negative, $f$ is invreasing. Now, suppose that $f(x)=f(y)$ for some $x$ and some $y$ such that $x<y$. Then $f$ is constant in $[x,y]$ and therefore $f'$ is $0$ there. But $f'$ only has one single zero.
              $endgroup$
              – José Carlos Santos
              Dec 16 '18 at 14:10




              $begingroup$
              SInce $f'(x)$ is always non-negative, $f$ is invreasing. Now, suppose that $f(x)=f(y)$ for some $x$ and some $y$ such that $x<y$. Then $f$ is constant in $[x,y]$ and therefore $f'$ is $0$ there. But $f'$ only has one single zero.
              $endgroup$
              – José Carlos Santos
              Dec 16 '18 at 14:10











              2












              $begingroup$

              Here is an approach if you want to avoid derivatives. For $x neq 0$, we have
              $$frac{x^3}{x^2 + 1} = frac{1}{frac{1}{x} + frac{1}{x^3}}.$$



              The function $1/x + 1/x^3$ for $x > 0$ is the sum of two strictly decreasing functions, hence itself strictly decreasing and therefore one to one with range $(0, infty)$. Hence $1/(1/x + 1/x^3)$ is strictly increasing and one to one with range $(0, infty)$. For $x < 0$, notice $1/x + 1/x^3$ is an odd function, and therefore a similar argument shows $1/(1/x + 1/x^3)$ is one to one with range $(-infty, 0)$.



              In fact, this shows that if you add back the point $x = 0$, the function is a bijection since the function is zero here.






              share|cite|improve this answer









              $endgroup$


















                2












                $begingroup$

                Here is an approach if you want to avoid derivatives. For $x neq 0$, we have
                $$frac{x^3}{x^2 + 1} = frac{1}{frac{1}{x} + frac{1}{x^3}}.$$



                The function $1/x + 1/x^3$ for $x > 0$ is the sum of two strictly decreasing functions, hence itself strictly decreasing and therefore one to one with range $(0, infty)$. Hence $1/(1/x + 1/x^3)$ is strictly increasing and one to one with range $(0, infty)$. For $x < 0$, notice $1/x + 1/x^3$ is an odd function, and therefore a similar argument shows $1/(1/x + 1/x^3)$ is one to one with range $(-infty, 0)$.



                In fact, this shows that if you add back the point $x = 0$, the function is a bijection since the function is zero here.






                share|cite|improve this answer









                $endgroup$
















                  2












                  2








                  2





                  $begingroup$

                  Here is an approach if you want to avoid derivatives. For $x neq 0$, we have
                  $$frac{x^3}{x^2 + 1} = frac{1}{frac{1}{x} + frac{1}{x^3}}.$$



                  The function $1/x + 1/x^3$ for $x > 0$ is the sum of two strictly decreasing functions, hence itself strictly decreasing and therefore one to one with range $(0, infty)$. Hence $1/(1/x + 1/x^3)$ is strictly increasing and one to one with range $(0, infty)$. For $x < 0$, notice $1/x + 1/x^3$ is an odd function, and therefore a similar argument shows $1/(1/x + 1/x^3)$ is one to one with range $(-infty, 0)$.



                  In fact, this shows that if you add back the point $x = 0$, the function is a bijection since the function is zero here.






                  share|cite|improve this answer









                  $endgroup$



                  Here is an approach if you want to avoid derivatives. For $x neq 0$, we have
                  $$frac{x^3}{x^2 + 1} = frac{1}{frac{1}{x} + frac{1}{x^3}}.$$



                  The function $1/x + 1/x^3$ for $x > 0$ is the sum of two strictly decreasing functions, hence itself strictly decreasing and therefore one to one with range $(0, infty)$. Hence $1/(1/x + 1/x^3)$ is strictly increasing and one to one with range $(0, infty)$. For $x < 0$, notice $1/x + 1/x^3$ is an odd function, and therefore a similar argument shows $1/(1/x + 1/x^3)$ is one to one with range $(-infty, 0)$.



                  In fact, this shows that if you add back the point $x = 0$, the function is a bijection since the function is zero here.







                  share|cite|improve this answer












                  share|cite|improve this answer



                  share|cite|improve this answer










                  answered Dec 16 '18 at 14:17









                  RileyRiley

                  1825




                  1825























                      1












                      $begingroup$

                      Suppose $f(a)=f(b)$: then
                      $$
                      a^3(b^2+1)=b^3(a^2+1)
                      $$

                      This implies
                      $$
                      a^3b^2-a^2b^3+a^3-b^3=0
                      $$

                      and so
                      $$
                      a^2b^2(a-b)+(a-b)(a^2+ab+b^2)=0
                      $$

                      Conclude.




                      If you know that $a^2+ab+b^2le0$ if and only if $a=b=0$, you're done.




                      With calculus,
                      $$
                      f'(x)=frac{3x^2(x^2+1)-2x^4}{(x^2+1)^2}
                      $$

                      The derivative only vanishes at $0$, so Rolle's theorem would only apply to an interval of the form $[a,b]$, with $a<0$ and $b>0$. Can $f(a)=f(b)$ in this case?






                      share|cite|improve this answer









                      $endgroup$


















                        1












                        $begingroup$

                        Suppose $f(a)=f(b)$: then
                        $$
                        a^3(b^2+1)=b^3(a^2+1)
                        $$

                        This implies
                        $$
                        a^3b^2-a^2b^3+a^3-b^3=0
                        $$

                        and so
                        $$
                        a^2b^2(a-b)+(a-b)(a^2+ab+b^2)=0
                        $$

                        Conclude.




                        If you know that $a^2+ab+b^2le0$ if and only if $a=b=0$, you're done.




                        With calculus,
                        $$
                        f'(x)=frac{3x^2(x^2+1)-2x^4}{(x^2+1)^2}
                        $$

                        The derivative only vanishes at $0$, so Rolle's theorem would only apply to an interval of the form $[a,b]$, with $a<0$ and $b>0$. Can $f(a)=f(b)$ in this case?






                        share|cite|improve this answer









                        $endgroup$
















                          1












                          1








                          1





                          $begingroup$

                          Suppose $f(a)=f(b)$: then
                          $$
                          a^3(b^2+1)=b^3(a^2+1)
                          $$

                          This implies
                          $$
                          a^3b^2-a^2b^3+a^3-b^3=0
                          $$

                          and so
                          $$
                          a^2b^2(a-b)+(a-b)(a^2+ab+b^2)=0
                          $$

                          Conclude.




                          If you know that $a^2+ab+b^2le0$ if and only if $a=b=0$, you're done.




                          With calculus,
                          $$
                          f'(x)=frac{3x^2(x^2+1)-2x^4}{(x^2+1)^2}
                          $$

                          The derivative only vanishes at $0$, so Rolle's theorem would only apply to an interval of the form $[a,b]$, with $a<0$ and $b>0$. Can $f(a)=f(b)$ in this case?






                          share|cite|improve this answer









                          $endgroup$



                          Suppose $f(a)=f(b)$: then
                          $$
                          a^3(b^2+1)=b^3(a^2+1)
                          $$

                          This implies
                          $$
                          a^3b^2-a^2b^3+a^3-b^3=0
                          $$

                          and so
                          $$
                          a^2b^2(a-b)+(a-b)(a^2+ab+b^2)=0
                          $$

                          Conclude.




                          If you know that $a^2+ab+b^2le0$ if and only if $a=b=0$, you're done.




                          With calculus,
                          $$
                          f'(x)=frac{3x^2(x^2+1)-2x^4}{(x^2+1)^2}
                          $$

                          The derivative only vanishes at $0$, so Rolle's theorem would only apply to an interval of the form $[a,b]$, with $a<0$ and $b>0$. Can $f(a)=f(b)$ in this case?







                          share|cite|improve this answer












                          share|cite|improve this answer



                          share|cite|improve this answer










                          answered Dec 16 '18 at 14:09









                          egregegreg

                          184k1486205




                          184k1486205






























                              draft saved

                              draft discarded




















































                              Thanks for contributing an answer to Mathematics Stack Exchange!


                              • Please be sure to answer the question. Provide details and share your research!

                              But avoid



                              • Asking for help, clarification, or responding to other answers.

                              • Making statements based on opinion; back them up with references or personal experience.


                              Use MathJax to format equations. MathJax reference.


                              To learn more, see our tips on writing great answers.




                              draft saved


                              draft discarded














                              StackExchange.ready(
                              function () {
                              StackExchange.openid.initPostLogin('.new-post-login', 'https%3a%2f%2fmath.stackexchange.com%2fquestions%2f3042639%2fproof-y-x3-x21-is-injective%23new-answer', 'question_page');
                              }
                              );

                              Post as a guest















                              Required, but never shown





















































                              Required, but never shown














                              Required, but never shown












                              Required, but never shown







                              Required, but never shown

































                              Required, but never shown














                              Required, but never shown












                              Required, but never shown







                              Required, but never shown







                              Popular posts from this blog

                              Plaza Victoria

                              In PowerPoint, is there a keyboard shortcut for bulleted / numbered list?

                              How to put 3 figures in Latex with 2 figures side by side and 1 below these side by side images but in...